feat(analisi): aggiunge una risoluzione agli appunti del 30/03/2023

main
parent cf2e3aabb0
commit f3777151d8

@ -134,12 +134,12 @@
\[ \tilde f = \system{ f(x) & \text{se } x \neq a, b, \\ \ell_a & \text{se } x = a, \\ \ell_b & \text{se } x = b.} \] \[ \tilde f = \system{ f(x) & \text{se } x \neq a, b, \\ \ell_a & \text{se } x = a, \\ \ell_b & \text{se } x = b.} \]
Allora vale che $\tilde f$ è continua in $\overline I$\footnote{Come Allora\footnote{Come
già riscontrato, vale un risultato ancora più forte: già riscontrato, vale un risultato ancora più forte:
data un'estensione $\tilde f$ di $f$ in $\overline I$, $\tilde f$ data un'estensione $\tilde f$ di $f$ in $\overline I$, $\tilde f$
è continua se e solo se i valori estesi sono esattamente i limiti è continua se e solo se i valori estesi sono esattamente i limiti
della funzione nei punti di $I \setminus \overline I$; e quindi della funzione nei punti di $I \setminus \overline I$; e quindi
l'estensione continua è ben definita, e unica del suo genere.}. l'estensione continua è ben definita, e unica del suo genere.} vale che $\tilde f$ è continua in $\overline I$.
\end{exercise} \end{exercise}
\begin{solution} \begin{solution}
@ -160,14 +160,32 @@
%TODO: dimostrare che se limite sinistro e destro coincidono, allora esiste il limite ed è lo stesso del limite sinistro e destro. %TODO: dimostrare che se limite sinistro e destro coincidono, allora esiste il limite ed è lo stesso del limite sinistro e destro.
\begin{exercise} \begin{exercise}
Si trovi un esempio di $f : X \to \RRbar$, dove, dato $\xbar$ punto Si trovi un esempio di funzione $f : X \to \RRbar$, dove, dato $\xbar$ punto
di accumulazione di $X$, $f(x) \tendsto{\xbar} \ell$, ma di accumulazione di $X$, $f(x) \tendsto{\xbar} \ell$, ma
$\exists (x_n) \subseteq X$ tale che $x_n \tendston \xbar$, ma $\exists \, (x_n) \subseteq X$ tale che $x_n \tendston \xbar$, ma
$f(x_n)$ non tende a $\ell$ per $n \to \infty$. $f(x_n)$ non tende a $\ell$ per $n \to \infty$.
\end{exercise} \end{exercise}
\begin{solution} \begin{solution}
Sia $f : \RR \to \RR$ tale che:
\[ f(x) = \system{0 & \text{se } x = 0, \\ 1 & \text{altrimenti}.}\]
Si consideri allora la successione $(x_n) \subseteq X$ tale che:
\[ x_n = \system{ 0 & \text{se } n \text{ è pari}, \\ \frac{1}{n} & \text{altrimenti}. } \]
Si mostra che $x_n \tendston 0$. Infatti, sia $I = [-\eps, \eps]$, con $\eps > 0$, un intorno di $0$.
Allora per $n > \frac{1}{\eps}$ vale che $x_n \in I$ (infatti $0$ vi appartiene sempre, e $0 < \frac{1}{n} < \eps$);
da cui si ricava proprio che $x_n \tendston 0$. \\
Chiaramente $f(x) \tendsto{0} 1$. È sufficiente mostrare allora che $f(x_n)$ non tende a $1$ per
$n \to \infty$. Si consideri la sottosuccessione $f(x_{2n})$: poiché $f(x_{2n}) = f(0) = 0$, la
sottosuccessione presa in considerazione è costante, e quindi $f(x_{2n}) \tendston 0$.
Anche la sottosuccessione $f(x_{2n + 1})$ è costante, e vale che $f(x_{2n + 1}) = f(\frac{1}{n}) = 1$,
e quindi $f(x_{2n+1}) \tendston 1$. Poiché allora il limite di $f(x_n)$, se esistesse, dovrebbe essere
uguale a quello di ambo le sottosuccessioni considerate, ed il limite è unico, $f(x_n)$ non ammette
limite, proprio come volevasi dimostrare.
\end{solution} \end{solution}
\begin{exercise} \begin{exercise}
@ -186,7 +204,7 @@
da $\xbar$, assurdo dal momento che $I(\xbar)$ non ne contiene uno da $\xbar$, assurdo dal momento che $I(\xbar)$ non ne contiene uno
per costruzione; ma $x$ non può neanche appartenere a $X \setminus J$, per costruzione; ma $x$ non può neanche appartenere a $X \setminus J$,
dacché in tal modo si può sempre costruire con errore a piacimento dacché in tal modo si può sempre costruire con errore a piacimento
un intorno più piccolo di $I(x)$ tale che sia disgiunto con $J$, un intorno più piccolo di $J$ tale che sia disgiunto con $I(x)$,
\Lightning. Dal momento che $\QQ$ è denso in $\RRbar$, si può allora \Lightning. Dal momento che $\QQ$ è denso in $\RRbar$, si può allora
sempre associare a $J(\xbar)$ un numero razionale $q$ al suo interno. sempre associare a $J(\xbar)$ un numero razionale $q$ al suo interno.
In questo modo si può costruire una funzione $f : X \to \QQ$, In questo modo si può costruire una funzione $f : X \to \QQ$,

Loading…
Cancel
Save